4
$\begingroup$

It is curious to know whether the following assertion is ture or not?

If $A-B$ and $B$ are copositive matrices (implying $A$ is copositive) of the same size, then $\rho(A)\ge \rho(B)$, where $\rho$ means the spectral radius.

For positive definite matrices class and nonnegative (entrywise) matrices class, this is obviously true.

$\endgroup$

2 Answers 2

4
$\begingroup$

The assertion is false. Here is how to construct a counterexample.

  1. Let $A = XX^T + Y + Y^T$ where $Y \ge 0$ (elementwise)
  2. Let $B = XX^T$

Then, by construction $A$ is a copositive matrix (sum of semidefinite plus symmetric nonnegative matrix), and $B$ is copositive too (because it is semidefinite). Moreover, $A-B$ is also copositive because it is just a symmetric nonnegative symmetric.

However, if you try the above recipe to construct $A$ and $B$, then you get the following counterexample (via Matlab again) very rapidly.

$ X = \begin{pmatrix} -1.8393& -0.9342\\\\ 1.7632 & 1.6479 \end{pmatrix}$

$Y = \begin{pmatrix} 1.9949& 2.0663 \\\\ 2.3393& 0.1889 \end{pmatrix} $

$A = \begin{pmatrix} 8.2456 & -0.3770\\\\ -0.3770& 6.2024 \end{pmatrix} $

$B = \begin{pmatrix} 4.2558 &-4.7826\\\\ -4.7826 &5.8247 \end{pmatrix}$

Here, we have $\rho(A) = 8.3130$ and $\rho(B) = 9.8867$.

$\endgroup$
2
  • $\begingroup$ Very nice construction. And this consideration did not appear in optimization-online.org/DB_HTML/2011/06/3057.html There is also a counterexample (i.e., Example 2.2) to my previous problem concerning the power of copositive matrices. $\endgroup$
    – Sunni
    Jun 27, 2011 at 11:29
  • $\begingroup$ Thanks! Also thanks for pointing me to an interesting paper. $\endgroup$
    – Suvrit
    Jun 27, 2011 at 14:20
1
$\begingroup$

It is interesting to consider the same question for matrices with the Perron-Frobenius property (that may have negative entries). The answer is: practically yes, but.

Practically Yes: If $A$,$B^{T}$ (or, $A^{T},B$) have the Perron-Frobenius property and $A \leq B$, then $\rho(A) \leq \rho(B)$.

But: If $A \leq C \leq B$ and $A$,$B^{T}$ have the Perron-Frobenius property, then $\rho(C)$ can fall below $\rho(A)$.

Both the theorem and an example for the second statement can be found in: Dimitrios Noutsos, On Perron–Frobenius property of matrices having some negative entries, Linear Algebra and its Applications 412 (2006) 132–153

$\endgroup$

Your Answer

By clicking “Post Your Answer”, you agree to our terms of service and acknowledge you have read our privacy policy.

Not the answer you're looking for? Browse other questions tagged or ask your own question.